Difference between revisions of "2020 AMC 10A Problems/Problem 21"

(Created page with "There exists a unique strictly increasing sequence of nonnegative integers <math>a_1 < a_2 < … < a_k</math> such that<cmath>\frac{2^{289}+1}{2^{17}+1} = 2^{a_1} + 2^{a_2} +...")
 
m (Solution 1)
(31 intermediate revisions by 16 users not shown)
Line 1: Line 1:
 +
{{duplicate|[[2020 AMC 12A Problems|2020 AMC 12A #19]] and [[2020 AMC 10A Problems|2020 AMC 10A #21]]}}
 +
 +
==Problem==
 +
 
There exists a unique strictly increasing sequence of nonnegative integers <math>a_1 < a_2 < … < a_k</math> such that<cmath>\frac{2^{289}+1}{2^{17}+1} = 2^{a_1} + 2^{a_2} + … + 2^{a_k}.</cmath>What is <math>k?</math>
 
There exists a unique strictly increasing sequence of nonnegative integers <math>a_1 < a_2 < … < a_k</math> such that<cmath>\frac{2^{289}+1}{2^{17}+1} = 2^{a_1} + 2^{a_2} + … + 2^{a_k}.</cmath>What is <math>k?</math>
  
 
<math>\textbf{(A) } 117 \qquad \textbf{(B) } 136 \qquad \textbf{(C) } 137 \qquad \textbf{(D) } 273 \qquad \textbf{(E) } 306</math>
 
<math>\textbf{(A) } 117 \qquad \textbf{(B) } 136 \qquad \textbf{(C) } 137 \qquad \textbf{(D) } 273 \qquad \textbf{(E) } 306</math>
 +
 +
== Solution 1 ==
 +
First, substitute <math>2^{17}</math> with <math>a</math>.
 +
Then, the given equation becomes <math>\frac{a^{17}+1}{a+1}=a^{16}-a^{15}+a^{14}...-a^1+a^0</math>.
 +
Now consider only <math>a^{16}-a^{15}</math>. This equals <math>a^{15}(a-1)=a^{15}*(2^{17}-1)</math>.
 +
Note that <math>2^{17}-1</math> equals <math>2^{16}+2^{15}+...+1</math>, since the sum of a geometric sequence is <math>\frac{a^n-1}{a-1}</math>.
 +
Thus, we can see that <math>a^{16}-a^{15}</math> forms the sum of 17 different powers of 2.
 +
Applying the same method to each of <math>a^{14}-a^{13}</math>, <math>a^{12}-a^{11}</math>, ... , <math>a^{2}-a^{1}</math>, we can see that each of the pairs forms the sum of 17 different powers of 2. This gives us <math>17*8=136</math>.
 +
But we must count also the <math>a^0</math> term.
 +
Thus, Our answer is <math>136+1=\boxed{\textbf{(C) } 137}</math>.
 +
 +
~seanyoon777
 +
 +
== Solution 2 ==
 +
(This is similar to solution 1)
 +
Let <math>x = 2^{17}</math>. Then, <math>2^{289} = x^{17}</math>.
 +
The LHS can be rewritten as <math>\frac{x^{17}+1}{x+1}=x^{16}-x^{15}+\cdots+x^2-x+1=(x-1)(x^{15}+x^{13}+\cdots+x^{1})+1</math>. Plugging <math>2^{17}</math> back in for <math>x</math>, we have <math>(2^{17}-1)(2^{15\cdot{17}}+2^{13\cdot{17}}+\cdots+2^{1\cdot{17}})+1=(2^{16}+2^{15}+\cdots+2^{0})(2^{15\cdot17}+2^{13\cdot17}+\cdots+2^{1\cdot17})+1</math>. When expanded, this will have <math>17\cdot8+1=137</math> terms. Therefore, our answer is <math>\boxed{\textbf{(C) } 137}</math>.
 +
 +
==Solution 3 (Intuitive)==
 +
Multiply both sides by <math>2^{17}+1</math> to get <cmath>2^{289}+1=2^{a_1} + 2^{a_2} + … + 2^{a_k} + 2^{a_1+17} + 2^{a_2+17} + … + 2^{a_k+17}.</cmath>
 +
 +
Notice that <math>a_1 = 0</math>, since there is a <math>1</math> on the LHS. However, now we have an extra term of <math>2^{18}</math> on the right from <math>2^{a_1+17}</math>. To cancel it, we let <math>a_2 = 18</math>. The two <math>2^{18}</math>'s now combine into a term of <math>2^{19}</math>, so we let <math>a_3 = 19</math>. And so on, until we get to <math>a_{18} = 34</math>. Now everything we don't want telescopes into <math>2^{35}</math>. We already have that term since we let <math>a_2 = 18 \implies a_2+17 = 35</math>. Everything from now on will automatically telescope to <math>2^{52}</math>. So we let <math>a_{19}</math> be <math>52</math>.
 +
 +
As you can see, we will have to add <math>17</math> <math>a_n</math>'s at a time, then "wait" for the sum to automatically telescope for the next <math>17</math> numbers, etc, until we get to <math>2^{289}</math>. We only need to add <math>a_n</math>'s between odd multiples of <math>17</math> and even multiples. The largest even multiple of <math>17</math> below <math>289</math> is <math>17\cdot16</math>, so we will have to add a total of <math>17\cdot 8</math> <math>a_n</math>'s. However, we must not forget we let <math>a_1=0</math> at the beginning, so our answer is <math>17\cdot8+1 = \boxed{\textbf{(C) } 137}</math>.
 +
 +
==Solution 4==
 +
Note that the expression is equal to something slightly lower than <math>2^{272}</math>. Clearly, answer choices <math>(D)</math> and <math>(E)</math> make no sense because the lowest sum for <math>273</math> terms is <math>2^{273}-1</math>. <math>(A)</math> just makes no sense. <math>(B)</math> and <math>(C)</math> are 1 apart, but because the expression is odd, it will have to contain <math>2^0=1</math>, and because <math>(C)</math> is <math>1</math> bigger, the answer is <math>\boxed{\textbf{(C) } 137}</math>.
 +
 +
~Lcz
 +
 +
== Solution 5 ==
 +
In order to shorten expressions, <math>\#</math> will represent <math>16</math> consecutive <math>0</math>s when expressing numbers. <br>
 +
<br>
 +
Think of the problem in binary. We have <br>
 +
<math>\frac{1\#0\#0\#0\#0\#0\#0\#0\#0\#0\#0\#0\#0\#0\#0\#0\#0\#1_2}{1\#1_2}</math> <br>
 +
Note that <br>
 +
<math>(2^{17} + 1) (2^0 + 2^{34} + 2^{68} + \cdots + 2^{272}) = 2^0(2^{17} + 1) + 2^{34}(2^{17} + 1) + 2^{68}(2^{17} + 1) + \cdots 2^{272}(2^{17} + 1)</math> <br>
 +
<math>= 1\#1\#1\#1\#1\#1\#1\#1\#1\#1\#1\#1\#1\#1\#1\#1\#1\#1_2</math> <br>
 +
and <br>
 +
<math>(2^{17} + 1) (2^{17} + 2^{51} + 2^{85} + \cdots + 2^{255}) = 2^{17}(2^{17} + 1) + 2^{51}(2^{17} + 1) + 2^{85}(2^{17} + 1) + \cdots 2^{255}(2^{17} + 1)</math> <br>
 +
<math>= 1\#1\#1\#1\#1\#1\#1\#1\#1\#1\#1\#1\#1\#1\#1\#1\#0_2</math> <br>
 +
<br>
 +
Since <br>
 +
<math>\phantom{=\ } 1\#1\#1\#1\#1\#1\#1\#1\#1\#1\#1\#1\#1\#1\#1\#1\#1\#1_2</math> <br>
 +
<math>-\ \phantom{1\#} 1\#1\#1\#1\#1\#1\#1\#1\#1\#1\#1\#1\#1\#1\#1\#1\#0_2</math> <br>
 +
<math>= 1\#0\#0\#0\#0\#0\#0\#0\#0\#0\#0\#0\#0\#0\#0\#0\#0\#1_2</math> <br>
 +
this means that<br>
 +
<math>(2^{17} + 1) (2^0 + 2^{34} + 2^{68} + \cdots + 2^{272}) - (2^{17} + 1) (2^{17} + 2^{51} + 2^{85} + \cdots + 2^{255}) = 2^{289}</math> <br>
 +
so <br>
 +
<math>\frac{2^{289}+1}{2^{17}+1} = (2^0 + 2^{34} + 2^{68} + \cdots + 2^{272}) - (2^{17} + 2^{51} + 2^{85} + \cdots + 2^{255})</math> <br>
 +
<math>= 2^0 + (2^{34} - 2^{17}) + (2^{68} - 2^{51}) + \cdots + (2^{272} - 2^{255})</math> <br>
 +
<br>
 +
Expressing each of the pairs of the form <math>2^{n + 17} - 2^n</math> in binary, we have <br>
 +
<math>\phantom{=\ } 1000000000000000000 \cdots 0_2</math> <br>
 +
<math>-\ \phantom{10000000000000000} 10 \cdots 0_2</math> <br>
 +
<math>= \phantom{1} 111111111111111110 \cdots 0_2</math> <br>
 +
or <br>
 +
<math>2^{n + 17} - 2^n = 2^{n + 16} + 2^{n + 15} + 2^{n + 14} + \cdots + 2^{n}</math> <br>
 +
This means that each pair has <math>17</math> terms of the form <math>2^n</math>. <br>
 +
<br>
 +
Since there are <math>8</math> of these pairs, there are a total of <math>8 \cdot 17 = 136</math> terms. Accounting for the <math>2^0</math> term, which was not in the pair, we have a total of <math>136 + 1 = \boxed{\textbf{(C) } 137}</math> terms. ~[[User:emerald_block|emerald_block]]
 +
 +
==Video Solution==
 +
https://youtu.be/Ozp3k2464u4
 +
 +
~IceMatrix
 +
 +
==Video Solution==
 +
https://www.youtube.com/watch?v=FsCOVzhjUtE&list=PLLCzevlMcsWNcTZEaxHe8VaccrhubDOlQ&index=3 ~ MathEx
 +
 +
==See Also==
 +
 +
{{AMC10 box|year=2020|ab=A|num-b=20|num-a=22}}
 +
{{AMC12 box|year=2020|ab=A|num-b=18|num-a=20}}
 +
{{MAA Notice}}

Revision as of 15:12, 2 July 2020

The following problem is from both the 2020 AMC 12A #19 and 2020 AMC 10A #21, so both problems redirect to this page.

Problem

There exists a unique strictly increasing sequence of nonnegative integers $a_1 < a_2 < … < a_k$ such that\[\frac{2^{289}+1}{2^{17}+1} = 2^{a_1} + 2^{a_2} + … + 2^{a_k}.\]What is $k?$

$\textbf{(A) } 117 \qquad \textbf{(B) } 136 \qquad \textbf{(C) } 137 \qquad \textbf{(D) } 273 \qquad \textbf{(E) } 306$

Solution 1

First, substitute $2^{17}$ with $a$. Then, the given equation becomes $\frac{a^{17}+1}{a+1}=a^{16}-a^{15}+a^{14}...-a^1+a^0$. Now consider only $a^{16}-a^{15}$. This equals $a^{15}(a-1)=a^{15}*(2^{17}-1)$. Note that $2^{17}-1$ equals $2^{16}+2^{15}+...+1$, since the sum of a geometric sequence is $\frac{a^n-1}{a-1}$. Thus, we can see that $a^{16}-a^{15}$ forms the sum of 17 different powers of 2. Applying the same method to each of $a^{14}-a^{13}$, $a^{12}-a^{11}$, ... , $a^{2}-a^{1}$, we can see that each of the pairs forms the sum of 17 different powers of 2. This gives us $17*8=136$. But we must count also the $a^0$ term. Thus, Our answer is $136+1=\boxed{\textbf{(C) } 137}$.

~seanyoon777

Solution 2

(This is similar to solution 1) Let $x = 2^{17}$. Then, $2^{289} = x^{17}$. The LHS can be rewritten as $\frac{x^{17}+1}{x+1}=x^{16}-x^{15}+\cdots+x^2-x+1=(x-1)(x^{15}+x^{13}+\cdots+x^{1})+1$. Plugging $2^{17}$ back in for $x$, we have $(2^{17}-1)(2^{15\cdot{17}}+2^{13\cdot{17}}+\cdots+2^{1\cdot{17}})+1=(2^{16}+2^{15}+\cdots+2^{0})(2^{15\cdot17}+2^{13\cdot17}+\cdots+2^{1\cdot17})+1$. When expanded, this will have $17\cdot8+1=137$ terms. Therefore, our answer is $\boxed{\textbf{(C) } 137}$.

Solution 3 (Intuitive)

Multiply both sides by $2^{17}+1$ to get \[2^{289}+1=2^{a_1} + 2^{a_2} + … + 2^{a_k} + 2^{a_1+17} + 2^{a_2+17} + … + 2^{a_k+17}.\]

Notice that $a_1 = 0$, since there is a $1$ on the LHS. However, now we have an extra term of $2^{18}$ on the right from $2^{a_1+17}$. To cancel it, we let $a_2 = 18$. The two $2^{18}$'s now combine into a term of $2^{19}$, so we let $a_3 = 19$. And so on, until we get to $a_{18} = 34$. Now everything we don't want telescopes into $2^{35}$. We already have that term since we let $a_2 = 18 \implies a_2+17 = 35$. Everything from now on will automatically telescope to $2^{52}$. So we let $a_{19}$ be $52$.

As you can see, we will have to add $17$ $a_n$'s at a time, then "wait" for the sum to automatically telescope for the next $17$ numbers, etc, until we get to $2^{289}$. We only need to add $a_n$'s between odd multiples of $17$ and even multiples. The largest even multiple of $17$ below $289$ is $17\cdot16$, so we will have to add a total of $17\cdot 8$ $a_n$'s. However, we must not forget we let $a_1=0$ at the beginning, so our answer is $17\cdot8+1 = \boxed{\textbf{(C) } 137}$.

Solution 4

Note that the expression is equal to something slightly lower than $2^{272}$. Clearly, answer choices $(D)$ and $(E)$ make no sense because the lowest sum for $273$ terms is $2^{273}-1$. $(A)$ just makes no sense. $(B)$ and $(C)$ are 1 apart, but because the expression is odd, it will have to contain $2^0=1$, and because $(C)$ is $1$ bigger, the answer is $\boxed{\textbf{(C) } 137}$.

~Lcz

Solution 5

In order to shorten expressions, $\#$ will represent $16$ consecutive $0$s when expressing numbers.

Think of the problem in binary. We have
$\frac{1\#0\#0\#0\#0\#0\#0\#0\#0\#0\#0\#0\#0\#0\#0\#0\#0\#1_2}{1\#1_2}$
Note that
$(2^{17} + 1) (2^0 + 2^{34} + 2^{68} + \cdots + 2^{272}) = 2^0(2^{17} + 1) + 2^{34}(2^{17} + 1) + 2^{68}(2^{17} + 1) + \cdots 2^{272}(2^{17} + 1)$
$= 1\#1\#1\#1\#1\#1\#1\#1\#1\#1\#1\#1\#1\#1\#1\#1\#1\#1_2$
and
$(2^{17} + 1) (2^{17} + 2^{51} + 2^{85} + \cdots + 2^{255}) = 2^{17}(2^{17} + 1) + 2^{51}(2^{17} + 1) + 2^{85}(2^{17} + 1) + \cdots 2^{255}(2^{17} + 1)$
$= 1\#1\#1\#1\#1\#1\#1\#1\#1\#1\#1\#1\#1\#1\#1\#1\#0_2$

Since
$\phantom{=\ } 1\#1\#1\#1\#1\#1\#1\#1\#1\#1\#1\#1\#1\#1\#1\#1\#1\#1_2$
$-\ \phantom{1\#} 1\#1\#1\#1\#1\#1\#1\#1\#1\#1\#1\#1\#1\#1\#1\#1\#0_2$
$= 1\#0\#0\#0\#0\#0\#0\#0\#0\#0\#0\#0\#0\#0\#0\#0\#0\#1_2$
this means that
$(2^{17} + 1) (2^0 + 2^{34} + 2^{68} + \cdots + 2^{272}) - (2^{17} + 1) (2^{17} + 2^{51} + 2^{85} + \cdots + 2^{255}) = 2^{289}$
so
$\frac{2^{289}+1}{2^{17}+1} = (2^0 + 2^{34} + 2^{68} + \cdots + 2^{272}) - (2^{17} + 2^{51} + 2^{85} + \cdots + 2^{255})$
$= 2^0 + (2^{34} - 2^{17}) + (2^{68} - 2^{51}) + \cdots + (2^{272} - 2^{255})$

Expressing each of the pairs of the form $2^{n + 17} - 2^n$ in binary, we have
$\phantom{=\ } 1000000000000000000 \cdots 0_2$
$-\ \phantom{10000000000000000} 10 \cdots 0_2$
$= \phantom{1} 111111111111111110 \cdots 0_2$
or
$2^{n + 17} - 2^n = 2^{n + 16} + 2^{n + 15} + 2^{n + 14} + \cdots + 2^{n}$
This means that each pair has $17$ terms of the form $2^n$.

Since there are $8$ of these pairs, there are a total of $8 \cdot 17 = 136$ terms. Accounting for the $2^0$ term, which was not in the pair, we have a total of $136 + 1 = \boxed{\textbf{(C) } 137}$ terms. ~emerald_block

Video Solution

https://youtu.be/Ozp3k2464u4

~IceMatrix

Video Solution

https://www.youtube.com/watch?v=FsCOVzhjUtE&list=PLLCzevlMcsWNcTZEaxHe8VaccrhubDOlQ&index=3 ~ MathEx

See Also

2020 AMC 10A (ProblemsAnswer KeyResources)
Preceded by
Problem 20
Followed by
Problem 22
1 2 3 4 5 6 7 8 9 10 11 12 13 14 15 16 17 18 19 20 21 22 23 24 25
All AMC 10 Problems and Solutions
2020 AMC 12A (ProblemsAnswer KeyResources)
Preceded by
Problem 18
Followed by
Problem 20
1 2 3 4 5 6 7 8 9 10 11 12 13 14 15 16 17 18 19 20 21 22 23 24 25
All AMC 12 Problems and Solutions

The problems on this page are copyrighted by the Mathematical Association of America's American Mathematics Competitions. AMC logo.png